Câu 37212 - Tự Học 365
Câu hỏi Vận dụng

Trong không gian với hệ tọa độ $Oxyz,$ cho hai điểm $A\left( { - 2;3;1} \right)$và $B\left( {5;{\rm{ }}6;{\rm{ }}2} \right)$. Đường thẳng $AB$cắt mặt phẳng $\left( {Oxz} \right)$ tại điểm $M$. Tính tỉ số $\dfrac{{AM}}{{BM}}$.


Đáp án đúng: a
Luyện tập khác

Phương pháp giải

- Gọi tọa độ của \(M,\) sử dụng điều kiện thẳng hàng của \(A,B,M\) tìm \(M\)

- Tính \(AB,AM\) và kết luận \(\dfrac{{AM}}{{BM}}\)

Xem lời giải

Lời giải của Tự Học 365

\(M \in \left( {Oxz} \right){\rm{ }} \Rightarrow M\left( {x{\rm{ ; 0 ; }}z} \right)\)

\(\overrightarrow {AB}  = \left( {7{\rm{ ; }}3{\rm{ ; }}1} \right){\rm{  }} \Rightarrow AB = \sqrt[{}]{{59}}\)

\(\overrightarrow {AM}  = \left( {x + 2{\rm{ ;}} - 3{\rm{ ; }}z - 1} \right)\)và

\(A,B,M\)thẳng hàng \( \Rightarrow \overrightarrow {AM}  = k.\overrightarrow {AB} {\rm{   }}\left( {k \in \mathbb{R}} \right)\)\( \Leftrightarrow \left\{ \begin{array}{l}x + 2 = 7k\\ - 3 = 3k\\z - 1 = k\end{array} \right. \Leftrightarrow \left\{ \begin{array}{l}x =  - 9\\ - 1 = k\\z = 0\end{array} \right.\)\( \Rightarrow M\left( { - 9{\rm{ ; 0 ; }}0} \right)\)

\(\overrightarrow {BM}  = \left( { - 14{\rm{ ;}} - 6{\rm{ ;}} - 2} \right) \Rightarrow BM = \sqrt[{}]{{236}} = 2\sqrt {59} \)

\(\overrightarrow {AM}  = \left( { - 7; - 3; - 1} \right) \Rightarrow AM = \sqrt {59} \)

\( \Rightarrow \dfrac{{AM}}{{BM}} = \dfrac{1}{2}\).

Đáp án cần chọn là: a

Toán Lớp 12